LSAT and Law School Admissions Forum

Get expert LSAT preparation and law school admissions advice from PowerScore Test Preparation.

User avatar
 Dave Killoran
PowerScore Staff
  • PowerScore Staff
  • Posts: 5852
  • Joined: Mar 25, 2011
|
#59773
Complete Question Explanation
(The complete setup for this game can be found here: lsat/viewtopic.php?t=26553)

The correct answer choice is (D)

When H and Y are assigned to bench 1, the VFJ super block has only two options: benches 2 and 3 or benches 3 and 4. Since that much information could be tough to juggle in your mind, why not make two quick hypotheticals showing both possibilities?

pt26_s98_g1_q4.png

In both possibilities the key to assigning the remaining variables is in the GW not-block. In possibility #1, when G is assigned to bench 4, W cannot be assigned to bench 4 and must instead be assigned to bench 3. In possibility #2, when G is assigned to bench 2, W cannot be assigned to bench 2 and must instead be assigned to bench 4. Possibility #1 proves that answer choice (D) is correct.
You do not have the required permissions to view the files attached to this post.

Get the most out of your LSAT Prep Plus subscription.

Analyze and track your performance with our Testing and Analytics Package.